Matrix representation for a transformation

Click For Summary
The linear transformation T: R3-->R2 is represented by the matrix [[0, 3, -1], [1, 4, 1]]. The transformation is onto because the matrix has two pivots, indicating a consistent system. However, it is not one-to-one due to linear dependence, as there are more columns than rows. A more intuitive approach to determining one-to-one status could involve identifying distinct vectors in R3 that map to the same output. Overall, the analysis confirms the correctness of the matrix and the conclusions about the transformation's properties.
fk378
Messages
366
Reaction score
0

Homework Statement


Consider the linear transformation T: R2-->R2, where
T(x1, x2, x3)= (3x2-x3, x1+4x2+x3)

a. Find a matrix which implements this mapping.
b. Is this transformation one-to-one? Is it onto? Explain.

Homework Equations


[T(x)]_B = ([T]_B) (x_B)


The Attempt at a Solution


The matrix that implements this mapping would be the representation ([T]_B). I think that (x_B) is the vector [x1, x2, x3] and that [T(x)]_B is (3x2-x3, x1+4x2+x3) relative to the {x1, x2, x3} basis. So then ([T]_B) must be the matrix:

0 3 -1
1 4 1

Row-reducing this matrix to echelon form gives 2 pivots, so the transformation is onto since the system is consistent, but it is not one-to-one because the system is linearly dependent.


Are all my thoughts correct for this problem?
 
Physics news on Phys.org
The matrix is correct. T:R3->R2, not R2->R2. Your conclusions are also correct. Though I would probably figure them out in a less abstract way than counting 'pivots'.
 
How would you come to the conclusions? Would you first just point out that there are more columns than rows so one-to-one is not a possibility?
 
You might, for instance, find two distinct vectors of R3 which map into the same thing.
 
Question: A clock's minute hand has length 4 and its hour hand has length 3. What is the distance between the tips at the moment when it is increasing most rapidly?(Putnam Exam Question) Answer: Making assumption that both the hands moves at constant angular velocities, the answer is ## \sqrt{7} .## But don't you think this assumption is somewhat doubtful and wrong?

Similar threads

  • · Replies 26 ·
Replies
26
Views
3K
  • · Replies 24 ·
Replies
24
Views
3K
  • · Replies 9 ·
Replies
9
Views
3K
  • · Replies 4 ·
Replies
4
Views
2K
  • · Replies 1 ·
Replies
1
Views
1K
  • · Replies 4 ·
Replies
4
Views
2K
Replies
3
Views
2K
  • · Replies 4 ·
Replies
4
Views
2K
  • · Replies 10 ·
Replies
10
Views
3K
Replies
4
Views
2K